Solve Algebra Expression: \frac{1}{\sqrt{\left( \frac{y}{x}\right)^2 +1}}

  • Thread starter Thread starter FrogPad
  • Start date Start date
  • Tags Tags
    Algebra
Click For Summary
The discussion focuses on simplifying the algebraic expression \(\frac{1}{\sqrt{\left( \frac{y}{x}\right)^2 +1}}\). The TI-89 calculator simplifies it to \(\frac{|x|}{\sqrt{x^2+y^2}}\), prompting a query about the simplification process. Key steps involve multiplying both the numerator and denominator by \(|x|\) and recognizing the relationship between the components of the expression. The discussion emphasizes understanding algebraic manipulations rather than just relying on calculators. Overall, the simplification process is clarified through algebraic principles and manipulation techniques.
FrogPad
Messages
801
Reaction score
0
I'm working with this expression and I do not understand how to simplify it by hand:

\frac{1}{\sqrt{\left( \frac{y}{x}\right)^2 +1}}

My TI-89 reduces it to:
\frac{|x|}{\sqrt{x^2+y^2}}

How is it doing this? This is not homework. I'm sure it would be acceptiable to just put the simplification down on paper... but if you would rather give hints, that's fine. Thanks :)

The original expression was taken from:
\cos \tan^{-1} \frac{y}{x}
 
Last edited:
Physics news on Phys.org
Under the squareroot you have the expression \frac{x^2}{y^2}+1

Remember that
\frac{a}{b}\pm\frac{c}{d}=\frac{ad\pm bc}{bd}

And also, remember that
\frac{1}{\frac{a}{b}}=\frac{b}{a}

Do you see know how your calculator did it?
 
:)

hehe

god my algebra is WEAK.
Thanks.
 
Or: multiply both numerator and denominator by |x|:
\frac{|x|}{|x|\sqrt{\frac{y^2}{x^2}+ 1}}= \frac{|x|}{\sqrt{x^2(\frac{y^2}{x^2+ 1)}}= \frac{|x|}{\sqrt{y^2+ x^2}}
 
Question: A clock's minute hand has length 4 and its hour hand has length 3. What is the distance between the tips at the moment when it is increasing most rapidly?(Putnam Exam Question) Answer: Making assumption that both the hands moves at constant angular velocities, the answer is ## \sqrt{7} .## But don't you think this assumption is somewhat doubtful and wrong?

Similar threads

Replies
6
Views
2K
  • · Replies 5 ·
Replies
5
Views
1K
  • · Replies 5 ·
Replies
5
Views
2K
  • · Replies 9 ·
Replies
9
Views
3K
Replies
5
Views
2K
  • · Replies 2 ·
Replies
2
Views
1K
  • · Replies 12 ·
Replies
12
Views
3K
Replies
7
Views
2K
Replies
2
Views
2K
Replies
3
Views
2K